LSAT and Law School Admissions Forum

Get expert LSAT preparation and law school admissions advice from PowerScore Test Preparation.

 jiyounglee
  • Posts: 25
  • Joined: Aug 10, 2016
|
#27758
Hello powerscore,

Can someone explain why D is correct and other answer choices are wrong?

Thank you!
User avatar
 Stephanie Oswalt
PowerScore Staff
  • PowerScore Staff
  • Posts: 811
  • Joined: Jan 11, 2016
|
#27759
Hi jiyounglee,

Thanks for your question. Generally speaking, we need a bit more input from you before we delve into a discussion of a particular LR question. Ultimately, it won't be us who are taking the test; it's you! :-) Our goal is to help you cultivate the analytical ability to approach these questions on your own, which is why you need to help us help you first.

Here's what I'd like you to do:
  • 1. Describe your approach to the stimulus. Did you understand the argument, if any, from a structural standpoint? What is the conclusion, and what evidence is the author using in support of that conclusion?

    2. Did you prephrase an answer to the question in the stem? If so, what was your prephrase?

    3. What exactly made the other answer choices particularly attractive? Did you use any question type-specific test (e.g. Assumption Negation Technique) to differentiate between them?
Thanks,

Stephanie
 jiyounglee
  • Posts: 25
  • Joined: Aug 10, 2016
|
#27768
1. Describe your approach to the stimulus. Did you understand the argument, if any, from a structural standpoint? What is the conclusion, and what evidence is the author using in support of that conclusion?

S:

Premise 1: alleged trend might not be real

Premise 2: After all, scientists disagree about it, some predicting over twice as much warming as others.

Intermediate conclusion: their predictions cannot be based on firm evidence

conclusion: it would be premature to act to halt the threatened "global warming trend"

W:

Premise 1: Most scientists consider discussions of accepted ideas boring, and prefer to argue about what is not known

Premise 2: According to the international science council, there is a consensus among reputable investigators that average global warming in the next century will be from 1.5 degrees to 4.5 degrees (specific data is used)

conclusion is not explicitly indicated here but I assume that conclusion of W is it would be not premature to act to halt the threatened global warming trend (opposite to S)

2. Did you prephrase an answer to the question in the stem? If so, what was your prephrase?

As this is method of reasoning question, I have prephrased as "arguing against S that there is a consensus based on specific data"

3. What exactly made the other answer choices particularly attractive? Did you use any question type-specific test (e.g. Assumption Negation Technique) to differentiate between them?

A seemed to be attractive because W has mentioned that "there is a consensus"

C was also attractive because it may mean conflicts between International Science Council (authority in W) vs. scientists (authority in S)

I am confused what is means by D because of "in perspective by emphasizing similarities"

I didn't use any question-type specific test because this is a method of reasoning question. I'm not sure if there is any test for method of reasoning.
 Adam Tyson
PowerScore Staff
  • PowerScore Staff
  • Posts: 5153
  • Joined: Apr 14, 2011
|
#27773
That's what we're looking for! With a look into your reasoning and your approach we can provide better feedback that should be more useful to you than just a general response.

First, I think your analysis of S's argument is sound and accurate. Nice work.

Next, I think you've begun to go a little astray in ascribing a conclusion to W. In a Method of Reasoning argument, you are looking at a question type that is in the "Prove" family, like a Must Be True question. That means you need to stick with what was in the stimulus and not bring in any outside information. Here, the conclusion that you have assumed W to be supporting is just that, an assumption on your part, an inference that you have drawn. It's a reasonable one, and probably true, but don't let that cloud your analysis of what actually happened.

I'm not sure about your prephrase here - it seems that maybe you have let your belief about what W might conclude influence you a little bit. Is W arguing against S's evidence? S says that scientists disagree about how much warming has occurred - does W dispute that? I don't think so. Rather, she focuses not on the disagreement (over how much) but on what they do agree on (warming is happening within some agreed-upon range). In the absence of a conclusion, I think it's fair to say that W isn't arguing at all, just presenting additional information. Sure, she might be about to make an argument, but maybe not. In any case, she didn't make one, so a more accurate prephrase might be that she presented new evidence that S may not have considered.

With that in mind, the answers that at first seemed attractive may now appear to be less so. A, for example - did W actually dispute that scientists disagree about the amount of warming? Nope - her claims are totally consistent with that idea, although she puts it into a larger context (they don't all agree on the amount, but they do agree that it fits within a certain range). In answer C, do we have battling authorities here? Does S cite someone who says one thing and then W cite someone else who says something in opposition to that? Nope. S cites scientists as a group and says that within the group there is disagreement about how much. W cites an organization that puts those disagreements in a larger context, without actually disagreeing with any particular scientist. No conflicting views here, just different context.

So what's this perspective thing? W is implicitly conceding that not every scientist agrees on the exact amount, but they do agree on a range - the perspective is larger, more global, about the scientific community rather than between individual scientists. While some may say 1.5 degrees and others say 4.5 degrees and still others fall somewhere in between, W tells us that they would pretty much all agree that it is no less and no more than those two extremes.

Take another look at what W says, and what she does not say, and see if that makes some sense.
 jiyounglee
  • Posts: 25
  • Joined: Aug 10, 2016
|
#27778
Hello Adam,

That makes much more sense!
Thank you so much!

Get the most out of your LSAT Prep Plus subscription.

Analyze and track your performance with our Testing and Analytics Package.